3
$\begingroup$

I am interested in Airy's equation $$\frac{\partial u}{\partial t}(t,x)=-\frac{\partial^3 u}{\partial x^3}(t,x)$$ on a bounded or semi-bounded domain, e.g. on $(-\infty,0)$. In order to obtain a group of isometries as propagator in $L^2(-\infty,0)$, two boundary conditions have to imposed - e.g., $u'(0)=0$ and either $u(0)=0$ or $u''(0)=0$. But then it seems that the spectrum of the operator is far too large, so I doubt that this operator does not generate a semigroup in $L^2(-\infty,0)$ at all, regardless of the imposed boundary conditions. Is it so? Has anybody ever proved well-posedness of the above equation in some reasonable function or distribution space?

$\endgroup$
4
  • 2
    $\begingroup$ On a bounded interval, $A=iD^3$ has self-adjoint realizations and then $e^{itA}$ is a unitary group. On a half line, $D^3$ has unequal deficiency indices $(2,1)$, so this approach won't work. $\endgroup$ Jan 13, 2016 at 18:46
  • $\begingroup$ On a bounded interval, the deficiency index is $3$, so you need this many boundary conditions (not two). $\endgroup$ Jan 13, 2016 at 18:54
  • $\begingroup$ @ChristianRemling Indeed! There are several works suggesting that two b.c. have to be imposed on the right and one on the left endpoints; but in these works I can never find a "clean" well-posedness theorem in a "nice" function space (ideally, a Sobolev or Lebesgue space, but I'd be happy even with one space from Triebel's zoo; ideally, by means of a unitary $C_0$-group). Remarkably, the equation on $\mathbb R$ is governed by a propagator that is just the convolution of an integral kernel based on Airy's function and the Fourier transform of the initial data, so a group might exist after all. $\endgroup$ Jan 13, 2016 at 20:04
  • $\begingroup$ Yes, on $\mathbb R$ the $L^2$ theory is straightforward: $A=iD^3$ is essentially self-adjoint on $C_0^{\infty}$, so again $e^{itA}$ is a unitary group. $\endgroup$ Jan 13, 2016 at 21:57

2 Answers 2

3
$\begingroup$

The operators $A=-\,\partial_x^3$ with domain $\mathscr D(A) = \left(H^3\cap H_0^2\right)(\mathbb R_-)$ and $A^\ast=\partial_x^3$ with domain $\mathscr D(A^\ast) = \left(H^3\cap H_0^1\right)(\mathbb R_-)$ are $m$-dissipative on $L^2(\mathbb R_-)$. This follows from $\Re\langle Au,u\rangle =0$ for $u\in\mathscr D(A)$ and $\Re\langle A^\ast v,v\rangle =-\,\frac12\,|v'(0)|^2\leq 0$ for $v\in\mathscr D(A^\ast)$. Therefore, $A$ and $A^\ast$ generate $C_0$-contraction semigroups on $L^2(\mathbb R_-)$, by the Lumer-Phillips theorem. Moreover, the semigroup $\{e^{tA}\}_{t\geq0}$ consists of isometries (though not of unitary operators for $t>0$, as $iA$ is not selfadjoint, see Christian's comment above).

One can also directly solve the initial-boundary value problem $$ \left\{ \enspace \begin{aligned} & u_t + u_{xxx} = f(t,x), & (t,x)\in \mathbb R_+ \times \mathbb R_-, \\ & u\bigr|_{x=0} = h_0(t), \enspace u_x\bigr|_{x=0} = h_1(t), \\ & u\bigr|_{t=0} = u_0(x) \end{aligned} \right. $$ (and similar for the operator $\partial_t-\partial_x^3$ with one in place of two boundary conditions), e.g., by using the Laplace transform with respect to $x$. See N. Hayashi, E. Kaikina, Nonlinear Theory of Pseudodifferential Equations on a Half-line, North-Holland, 2004, which contains a chapter on linear problems.

$\endgroup$
3
$\begingroup$

This is a very interesting question and I do not know the answer. I would start at something like

N. A. Larkin, Correct initial boundary value problems for dispersive equations, J. Math. Anal. Appl. 344 (2008), no. 2, 1079--1092.

$\endgroup$

Your Answer

By clicking “Post Your Answer”, you agree to our terms of service and acknowledge you have read our privacy policy.

Not the answer you're looking for? Browse other questions tagged or ask your own question.